Livro de Urantia

Grupo de Aprendizes da Informação Aberta

Contato

Índice Superior

Arquivos de Impressão: Tamanho A4 (pdf), Tamanho A5 (pdf), Texto (txt).

Prova de Física para o ITA em 2016
Concurso de admissão para o
Instituto Tecnológico de Aeronáutica

Sumário

Prova de Física do ITA para 2016
    1.1  Questão 1 - Hidrodinâmica
    1.2  Questão 2 - Mecânica, Estática
    1.3  Questão 3 - Mecânica, Cinemática
    1.4  Questão 4 - Mecânica, Ondulatória
    1.5  Questão 5 - Mecânica, Dinâmica
    1.6  Questão 6 - Mecânica, Força centrífuga, Atrito, Torque
    1.7  Questão 7 - Gravitação
    1.8  Questão 8 - Hidrodinâmica
    1.9  Questão 9 - Termologia, Ondulatória e Mecânica
    1.10  Questão 10 - Dinâmica, Ondulatória, Pêndulo
    1.11  Questão 11 - Eletricidade e Magnetismo
    1.12  Questão 12 - Hidrodinâmica, tubo de Pitot
    1.13  Questão 13 - Teoria dos Gases
    1.14  Questão 14 - Hidrodinâmica, Empuxo
    1.15  Questão 15 - Mecânica, Atrito Estático, Torque
    1.16  Questão 16 - Ondulatória, Cordas
    1.17  Questão 17 - Óptica, Refração
    1.18  Questão 18 - Circuitos Elétricos
    1.19  Questão 19 - Indução Magnética
    1.20  Questão 20 - Efeito Doppler Relativístico
    1.21  Questão 21 - Circuitos Elétricos
    1.22  Questão 22 - Mecânica, Cinemática
    1.23  Questão 23 - Mecânica, Hidrodinâmica
    1.24  Questão 24 - Ondulatória, Acústica
    1.25  Questão 25 - Mecânica, Atrito
    1.26  Questão 26 - Termologia, Conversão de Energia
    1.27  Questão 27 - Resistência Elétrica Variável
    1.28  Questão 28 - Mecânica, Choques Elásticos e Eletrostática
    1.29  Questão 29 - Mecânica, Movimento Harmônico Simples
    1.30  Questão 30 - Teoria dos Circuitos

1  Prova de Física do ITA para 2016

     Quando precisar use os seguintes valores para as constantes:

1.1  Questão 1 - Hidrodinâmica

     Considere um corpo esférico de raio r totalmente envolvido por um fluido de viscosidade η com velocidade média v. De acordo com a lei de Stokes, para baixas velocidades, esse corpo sofrerá a ação de uma força de arrasto viscoso dada por F = −6 πr ηv. A dimensão de η é dada por

     Resolução

     F = −6 πr ηv

     [F] = [η] [r] [V]

     M L T−2 = [η] L L T−1

    

[η] = M L−1 T−1

    

μ[η] = kg . m−1 s−1

     Resposta: E

1.2  Questão 2 - Mecânica, Estática

     Três barras de peso desprezível, articuladas nos pinos P, Q e R, constituem uma estrutura vertical em forma de triângulo isósceles com 6,0 m de base e 4,0 m de altura, que sustenta uma massa M suspensa em Q em equilíbrio estático. O pino P também é articulado no seu apoio fixo, e o pino R apoia-se verticalmente sobre o rolete livre.

    

     Sendo de 1,5 ×104 N e 5,0 ×103 N os respectivos valores máximos das forças de tração e compressão suportáveis por qualquer das barras, o máximo valor possível para M é de

     Resolução

     1) Forças atuantes em Q:

    

     Para o equilíbrio do ponto Q:

     2F cos 37o = P

     2F . 0,80 = M . 10 ⇒F = [ 10M/ 1,6] (SI)

     2) A barra [PQ] está sendo comprimida pela força de intensidade F:

    

     Fmáx = [( 10M1)/ 1,6] = 5,0 . 103

     M1 = 8,0 . 102 kg

     3)

    

     A força resultante entre a força normal FN e a força Fh aplicada na haste [PR] deve ser igual à força de compressão F aplicada na haste [RQ].

     Fh = F cos 53o

     Fh = [ 10M/ 1,6] 0,6 (SI)

    

Fh = [ 6,0M/ 1,6] (SI)

     A haste [PR] estará sendo tracionada pela força de intensidade Fh:

    

     Fmáx = [( 6,0M2)/ 1,6] = 1,5 . 104 (SI)

    

M2 = 4,0 . 103 kg

     Para que nenhuma das barras se rompa, devemos usar o menor valor entre M1 e M2:

    

Mmáx = 8,0 . 102 kg

     Resposta: C

1.3  Questão 3 - Mecânica, Cinemática

     No sistema de sinalização de trânsito urbano chamado de "onda verde", há semáforos com dispositivos eletrônicos que indicam a velocidade a ser mantida pelo motorista para alcançar o próximo sinal ainda aberto. Considere que de início o painel indique uma velocidade de 45 km/h. Alguns segundos depois ela passa para 50 km/h e, finalmente, para 60 km/h. Sabendo que a indicação de 50 km/h no painel demora 8,0 s antes de mudar para 60 km/h, então a distância entre os semáforos é de

     Resolução

     Consideremos dois semáforos, S1 e S2, separados por uma distância D.

    

     Um primeiro carro, A, passa por S1 e deverá manter uma velocidade escalar de 45km/h para pegar S2 aberto, gastando um tempo T1.

     Um segundo carro, B, passa por S1 e deverá manter uma velocidade escalar de 50km/h para pegar S2 aberto, gastando um tempo T2.

     Portanto:

     T1 − T2 = 8,0s.

     ∆s = Vt (Movimento Uniforme)

     D = [ 45/ 3,6] . T1 = [ 50/ 3,6] . T2

     T1 = [ 3,6D/ 45] e T2 = [ 3,6D/ 50]

     [ 3,6D/ 45] − [ 3,6D/ 50] = 8,0 (SI)

     2D = [ 7200/ 3,6]

     D = 1 000m

    

D = 1,0km

     Resposta: D

1.4  Questão 4 - Mecânica, Ondulatória

     Um bloco de massa m encontra-se inicialmente em repouso sobre uma plataforma apoiada por uma mola, como visto na figura.

    

     Em seguida, uma pessoa de massa M sobe na plataforma e ergue o bloco até uma altura h da plataforma, sendo que esta se desloca para baixo até uma distância d. Quando o bloco é solto das mãos, o sistema (plataforma + pessoa + mola) começa a oscilar e, ao fim da primeira oscilação completa, o bloco colide com a superfície da plataforma num choque totalmente inelástico. A razão entre a amplitude da primeira oscilação e a da que se segue após o choque é igual a

     Resolução

     1) O acréscimo de deformação da mola é provocado pelo peso da pessoa:

     M g = k d

    

⇒ k = [ Mg/ d]

     2) Na posição de equilíbrio:

     Fe = (M + m) g

     Quando m é abandonada, a aceleração adquirida pela pessoa é dada por:

     Fe − M g = M a

     (M + m) g − M g = M a

    

a = [ m g/ M]

     Esta aceleração é a aceleração máxima do MHS (Movimento Harmônico Simples) e é dada por:

     a = amáx = [ m g/ M] = ω2 A1 (1)

     Por outro lado: k = M ω2 ⇒ [ M g/ d] = M ω2

    

ω2 = [ g/ d] (2)

     Substituindo-se (2) em (1), vem:

     [ m g/ M] = [ g/ d] . A1

    

⇒ A1 = [ m d/ M]

     3) A velocidade do bloco m no instante da colisão é dada por:

     V12 = 0 + 2 g h ⇒ V1 = √{2 g h}

     4) No instante da colisão, a plataforma completou sua oscilação e voltou ao repouso. Usando a conservação da quantidade de movimento:

     Qf = Qi

     (M + m) V2 = m V1

     (M + m) V2 = m √{2 g h}

     V2 = [ m/( M + m)] √{2 g h}

     5) A nova posição de equilíbrio corresponde à posição da plataforma no instante em que o bloco m foi abandonado e portanto V2 será a velocidade máxima do novo MHS.

     6) A nova pulsação será dada por:

     k = (M + m) ω12

     ω12 = [ k/( M + m)] ⇒ ω1 = √{[ k/( M + m)]}

     7) A nova amplitude de oscilação A2 é dada por:

     V2 = ω1 A2

     [ m/( M + m)] √{2 g h} = √{[ k/( M + m)] A2}

     A2 = [( m √{2 g h})/( M + m)] √{[( M + m)/ k]}

    

A2 = [ m/( M + m)] √{[( 2 g h (M + m))/ k]}

     A2 = [ m/( M + m)] √{[( 2 g h (M + m))/( [ M g/ d])]}

     A2 = [ m/( M + m)] √{[( 2 d h (M + m))/ M]}

     [( A1)/( A2)] = [ m d/ M] . [( M + m)/ m] √{[ M/( 2 d h (M + m))]}

     [( A1)/( A2)] = [( (M + m) d)/ M] √{[ M/( 2 d h (M + m))]}

     [( A1)/( A2)] = √{[( (M + m))/ M] [ d/ 2 h]}

     Resposta: SEM RESPOSTA

1.5  Questão 5 - Mecânica, Dinâmica

     A partir do repouso, um foguete de brinquedo é lançado verticalmente do chão, mantendo uma aceleração constante de 5,00 m/s2 durante os 10,0 primeiros segundos. Desprezando a resistência do ar, a altura máxima atingida pelo foguete e o tempo total de sua permanência no ar são, respectivamente, de

     Resolução

     1) Cálculo da altura após 10,0s:

     H = H0 + V0 t + [( γ)/ 2] t2

     H = 0 + 0 + [ 5,00/ 2] . 100 (m)

    

⇒ H1 = 250m

     2) Cálculo da velocidade escalar após 10,0s:

     V = V0 + γt

     H = 0 + 5,0 . 10,0 (m/s)

    

⇒ V1 = 50,0m/s

     3) Cálculo da altura máxima atingida:

     V2 = V12 + 2 γ∆H

     0 = 2500 + 2 (−10,0) (Hmáx − 250)

     20,0 (Hmáx − 250) = 2500

     Hmáx = 125 + 250 (m)

    

Hmáx = 375m

     4) Cálculo do tempo sob ação da gravidade:

     h = H1 + V1 t + [( γ)/ 2] t2

     0 = 250 + 50,0 T1 − 5,0 T12

     5,0 T12 − 50,0 T1 − 250 = 0

     T12 − 10,0 T1 − 50,0 = 0

     T1 = [( 10,0 ± √{100 + 200})/ 2] (s)

    

T1 ≅ 13,7s

     5) O tempo total:

     T = T1 + 10,0s

    

⇒ T = 23,7s

     Resposta: A

1.6  Questão 6 - Mecânica, Força centrífuga, Atrito, Torque

     Um caminhão baú de 2,00m de largura e centro de gravidade a 3,00m de chão percorre um trecho de estrada em curva com 76,8m de raio. Para manter a estabilidade do veículo neste trecho, sem derrapar, sua velocidade não deve exceder a

     Resolução

    

     1) Na iminência de tombamento, o somatório dos torques em relação ao CG é nulo:

     Fat . 3,00 = FN . 1,00

     Fat . 3,00 = m . 10

    

Fat = [ 10m/ 3,00]

     2) A força de atrito faz o papel de resultante centrípeta:

     Fat = [( m V2)/ R]

     [ 10m/ 3,00] = [( m V2)/ 76,8]

     V2 = 256 (SI)

    

⇒ V = 16,0m/s

     Resposta: C

1.7  Questão 7 - Gravitação

     Considere duas estrelas de um sistema binário em que cada qual descreve uma órbita circular em torno do centro de massa comum. Sobre tal sistema são feitas as seguintes afirmações:

     I. O período de revolução é o mesmo para as duas estrelas.

     II. Esse período é função apenas da constante gravitacional, da massa total do sistema e da distância entre ambas as estrelas.

     III. Sendo R1 e R2 os vetores posição que unem o centro de massa do sistema aos respectivos centros de massa das estrelas, tanto R1 como R2 varrem áreas de mesma magnitude num mesmo intervalo de tempo.

     Assinale a alternativa correta.

     Resolução

     I) (V) As estrelas estão sempre alinhadas com o centro de massa e portanto terão a mesma velocidade angular e o mesmo período de translação.

     II) (V)

    

     d = R1 + R2

     1) Posição do centro de massa:

     R1 = [( M . 0 + m . (R1 + R2))/( M + m)]

    

R1 = [ m d/( M + m)]

     2) F = Fcp

     [ G M m/( d2)] = M . ω2 . [ m d/( M + m)]

     [( G (M + m))/( d2)] = ω2 d

     ω2 = [( G (M + m))/( d3)] = ( [( 2 π)/ T] )2

     [( 4 π2)/( T2)] = [( G (M + m))/( d3)]

     T2 = [( 4 π2 d3)/( G (M + m))]

    

⇒ T = 2 π√{[( d3)/( G (M + m))]}

     T só depende de G, de (M + m) e de d.

     III) (F)

     Para o mesmo intervalo de tempo, o vetor posição de módulo maior varre área maior.

     Resposta: D

1.8  Questão 8 - Hidrodinâmica

     Um cubo de peso P1, construído com um material cuja densidade é ρ1, dispõe de uma região vazia em seu interior e, quando inteiramente imerso em um líquido de densidade ρ2, seu peso reduz-se a P2. Assinale a expressão com o volume da região vazia deste cubo.

     Resolução

     1) Cálculo de volume total V:

     P2 = P1 − E

     P2 = P1 − ρ2 V g

     ρ2 V g = P1 − P2

    

V = [( P1 − P2)/( ρ2 g)]

     2) Volume do material:

     P1 = ρ1 V1 g

    

V1 = [( P1)/( ρ1 g)]

     3) Cálculo do volume da parte vazia:

     V0 = V − V1

    

V0 = [( P1 − P2)/( ρ2 g)] − [( P1)/( ρ1 g)]

     Resposta: A

1.9  Questão 9 - Termologia, Ondulatória e Mecânica

     Um pêndulo simples é composto por uma massa presa a um fio metálico de peso desprezível. A figura registra medidas do tempo T em segundos, para 10 oscilações completas e seguidas do pêndulo ocorridas ao longo das horas do dia, t.

    

     Considerando que neste dia houve uma variação térmica total de 20oC, assinale o valor do coeficiente de dilatação térmica do fio deste pêndulo.

     Resolução

     Vamos considerar na resolução que o trecho do enunciado que diz "considerando que neste dia houve uma variação térmica total de 20oC" refira-se à máxima diferença de temperaturas verificada nesse dia, o que ocorreu entre 6h e 18h, segue-se que:

     (I)

     10 T2 − 10 T1 = 80,5 − 80,0

    

T2 − T1 = 5,0 . 10−2s (1)

     (II)

     T2 = 2 π√{[( L2)/ g]}

     ⇒ T2 = 2 π√{[( L1 (1 + α∆θ))/ g]}

     T1 = 2 π√{[( L1)/ g]}

     Logo: [( T2)/( T1)] = [( 2 π√{[( L1 (1 + α∆θ))/ g]})/( 2 π√{[( L1)/ g]})]

     Com ∆θ = 20oC, vem:

     [( T2)/( T1)] = √{1 + α20} ⇒

    

T2 = √{1 + α20} . T1 (2)

     (III) (2) em (1) e lembrando-se de que

     T1 = [ 80/ 10]s = 8,0 s, vem:

     √{1 + α20} . T1 − T1 = 5,0 . 10−2

     T1 (√{1 + α20} − 1) = 5,0 . 10−2

     8,0 (√{1 + α20} − 1) = 5,0 . 10−2

     √{1 + α20} = 1,00625

     Da qual:

     α ≅ 6 . 10−4oC−1

     Resposta: C

1.10  Questão 10 - Dinâmica, Ondulatória, Pêndulo

     Um pêndulo simples oscila com uma amplitude máxima de 60o em relação à vertical, momento em que a tensão no cabo é de 10 N. Assinale a opção com o valor da tensão no ponto em que ele atinge sua velocidade máxima.

     Resolução

     I)

    

cos60o = [( L − h)/ L]

     [ L/ 2] = L − h

     Da qual:

    

h = [ L/ 2]

     II) A resultante centrípeta no ponto A é nula, já que a velocidade nesse ponto é nula. Logo:

     Pn = TA ⇒ P cos60o = TA ⇒ P . [ 1/ 2] = 10

     P = 20 N

     III) Conservação de energia mecânica:

     EmB = EmA (referencial em B)

     [( m VB2)/ 2] = m g h ⇒ m VB2 = 2 P [ L/ 2]

     m VB2 = 2 . 20 [ L/ 2]

    

⇒ m VB2 = 20 L

     IV) O ponto B é o local da trajetória em que a velocidade tem intensidade máxima. Em B:

     TB − P = FcpB

     TB − P = [( m VB2)/ L] ⇒ T − 20 = [ 20 L/ L]

     T − 20 = 20

    

⇒ T = 40 N

     Resposta: D

1.11  Questão 11 - Eletricidade e Magnetismo

     Um líquido condutor (metal fundido) flui no interior de duas chapas metálicas paralelas, interdistantes de 2,0 cm, formando um capacitor plano, conforme a figura. Toda essa região interna está submetida a um campo homogêneo de indução magnética de 0,01 T, paralelo aos planos das chapas, atuando perpendicularmente à direção da velocidade do escoamento.

    

     Assinale a opção com o módulo dessa velocidade quando a diferença de potencial medida entre as placas for de 0,40 mV.

     Resolução

     Vamos supor que o líquido condutor contenha partículas eletrizadas que estejam deslocando-se com a mesma velocidade de escoamento do fluido.

    

     O movimento das partículas é retilíneo e uniforme. Assim, a força magnética e a força elétrica se equilibram.

     1. Cálculo do módulo do campo elétrico entre as placas P1 e P2:

     E.d = U

     E = [ U/ d] = [( 4,0 . 10−4 V)/( 2,0 . 10−2 m)] ⇒ E = 2,0 . 10−2 V/m

     2. Cálculo do módulo da velocidade:

     |Fm| = |Fe|

     |q|.V.B = |q|.E

     V = [ E/ B] ⇒ V = [( 2,0 . 10−2)/( 1,0 . 10−2)] (m/s)

    

V = 2,0 m/s

     Resposta: D

1.12  Questão 12 - Hidrodinâmica, tubo de Pitot

     Um estudante usa um tubo de Pitot esquematizado na figura para medir a velocidade do ar em um túnel de vento. A densidade do ar é igual a 1,2 kg/m3 e a densidade do líquido é 1,2 ×104 kg/m3, sendo h = 10 cm.

    

     Nessas condições a velocidade do ar é aproximadamente igual a

     Resolução

     A pressão hidrostática do líquido é equilibrada pela pressão dinâmica do ar:

     μL g h = [( μar Var2)/ 2]

     1,2 . 104 . 10 . 0,10 = [ 1,2/ 2] Var2

     Var2 = 2,0 . 104 (SI)

    

Var = 1,4 . 102 m/s

     Resposta: C

1.13  Questão 13 - Teoria dos Gases

     Balão com gás Hélio inicialmente a 27oC de temperatura e pressão de 1,0 atm, a mesma do ar externo, sobe até o topo de uma montanha, quando o gás se resfria a −23oC e sua pressão reduz-se a 0,33 de atm, também a mesma do ar externo. Considerando invariável a aceleração da gravidade na subida, a razão entre as forças de empuxo que atuam no balão nestas duas posições é

     Resolução

     Da Equação de Clapeyron, obtemos a densidade μ do ar no alto da montanha.

     pV = nRT

     pV = [ m/ M] RT (M = massa molar média do ar)

     pV = [( μV)/ M] RT

     μ = [ Mp/ RT]

     De forma análoga, a densidade inicial μ0 do ar é dada por:

     μ0 = [( M p0)/( R T0)]

     A intensidade E da força de empuxo sobre o balão é dada por:

     E = μV g

     E = [ M p/ R T] V g (1)

     E = μ0 V0 g

     E0 = [( M p0)/( R T0)] V0 g (2)

     Dividindo a equação (1) pela equação (2), temos:

     [ E/( E0)] = [( [ M p V/ R T] g)/( [( M p0 V0)/( R T0)] g)]

     [ E/( E0)] = [( [ p V/ T])/( [( p0 V0)/( T0)])]

     Da equação geral dos gases perfeitos, temos:

     [ p V/ T] = [( p0 V0)/( T0)]

     Portanto:

     [ E/( E0)] = 1

     Resposta: C

1.14  Questão 14 - Hidrodinâmica, Empuxo

     Um corpo flutua estavelmente em um tanque contendo dois líquidos imiscíveis, um com o dobro da densidade do outro, de tal forma que as interfaces líquido/líquido e líquido/ar dividem o volume do corpo exatamente em três partes iguais. Sendo completamente removido o líquido mais leve, qual proporção do volume do corpo permanece imerso no líquido restante?

     Resolução

    

     Na situação I:

     E = P

     2 ρ[ V/ 3] g + ρ[ V/ 3] g = P

     ρV g = P (1)

     Na situação II:

     E = P

     2 ρVi g = P (2)

     Comparando-se (1) e (2), vem:

     ρV g = 2 ρVi g

    

Vi = [ V/ 2]

     Resposta: A

1.15  Questão 15 - Mecânica, Atrito Estático, Torque

     A figura mostra uma placa fina de peso P dobrada em ângulo reto e disposta sobre uma esfera fixa de raio a.

    

     O coeficiente de atrito mínimo entre estes objetos para que a placa não escorregue é

     Resolução

    

     Iminência de escorregar:

     Fat = μFN

     F′at = μF′N

     1) Condição de força resultante nula:

     Fat = F′N

     μFN = F′N

     F′at + FN = P

     μFat + [( Fat)/( μ)] = P

     Fat + (μ+ [ 1/( μ)]) = P (1)

     2) Condição de torque nulo em relação ao ponto C:

     [ P/ 2] . R = F′at . R + Fat . R

     F′at + Fat = [ P/ 2]

     μFat + Fat = [ P/ 2]

     Fat (μ+ 1) = [ P/ 2] (2)

     [ (1)/ (2)] : [( μ+ [ 1/( μ)])/( μ+ 1)] = 2

     ⇒ μ+ [ 1/( μ)] = 2 μ+ 2

     μ2 + 1 = 2 μ2 + 2 μ⇒ μ2 + 2 μ− 1 = 0

     μ = [( −2 ± √{4 + 4})/ 2] ⇒ μ = [( −2 + 2 √2)/ 2]

    

μ = √2 − 1

     Obs.: Admitindo-se que a barra foi dobrada ao meio.

     Resposta: C

1.16  Questão 16 - Ondulatória, Cordas

     Uma corda de cobre, com seção de raio rC, está submetida a uma tensão T. Uma corda de ferro, com seção de raio rF, de mesmo comprimento e emitindo ondas de mesma frequência que a do cobre, está submetida a uma tensão T/3. Sendo de 1,15 a razão entre as densidades do cobre e do ferro, e sabendo que ambas oscilam no modo fundamental, a razão rC/rF é igual a

     Resolução

     (I) A frequência fundamental f de uma corda cilíndrica de comprimento L e raio r, submetida a uma força de tração T, é calculada pela Equação de Lagrange-Helmholtz.

     f = [ 1/ 2 L] √{[ T/( ρ)]} (1)

     Em que ρ é a densidade linear da corda ( ρ = [ m/ L] ).

     (II) Sendo μ a densidade volumétrica da corda, supostamente referida no enunciado, tem-se:

     μ = [ m/ Vol] = [ m/( πr2 L)] ⇒ μ = [( ρ)/( πr2)]

     Da qual: ρ = πμr2 (2)

     (III) (2) em (1):

     f = [ 1/ 2 L] √{[ T/( πμr2)]}

     (IV) No caso, fC = fF, logo:

     [ 1/( 2 LC)] √{[( TC)/( πμC rC2)]} = [ 1/( 2 LF)] √{[( TF)/( πμF rF2)]}

     Sendo LC = LF, TC = T, TF = [ T/ 3] e

     [( μC)/( μF)] = 1,15 ou μC = 1,15 μF, vem:

     [ T/( 1,15 μF rC2)] = [ T/( 3 μF rF2)] ⇒ ( [( rC)/( rF)] )2 = [ 3/ 1,15]

     Da qual:

    

[( rC)/( rF)] ≅ 1,6

     Resposta: D

1.17  Questão 17 - Óptica, Refração

     Um tubo de fibra óptica é basicamente um cilindro longo e transparente, de diâmetro d e índice de refração n. Se o tubo é curvado, parte dos raios de luz pode escapar e não se refletir na superfície interna do tubo.

    

     Para que haja reflexão total de um feixe de luz inicialmente paralelo ao eixo do tubo, o menor raio de curvatura interno R (ver figura) deve ser igual a

     Resolução

     O esquema refere-se à situação de maior possibilidade de emergência do raio de luz da fibra óptica para o ar.

    

     O seno do ângulo limite L para o dioptro fibra-ar é dado por:

sen L = [( nar)/ n] = [ 1/ n]

     A menor incidência interna na face FB da fibra ocorre para o raio de luz que se propaga sobre a reta [AB]. Do triângulo ABC, temos:

sen i = [ R/( (R + d))]

     Para que ocorra reflexão total em B, a condição é i > L.

     Portanto: sen i > sen L

     [ R/( (R + d))] > [ 1/ n]

    

R > [ d/( (n − 1))]

     Resposta: C

1.18  Questão 18 - Circuitos Elétricos

     No circuito da figura há três capacitores iguais, com C = 1 000 μF, inicialmente descarregados. Com as chaves (2) abertas e as chaves (1) fechadas, os capacitores são carregados. Na sequência, com as chaves (1) abertas e as chaves (2) fechadas, os capacitores são novamente descarregados e o processo se repete.

    

    

     Com a tensão no resistor R variando segundo o gráfico da figura, a carga transferida pelos capacitores em cada descarga é igual a

     Resolução

     1. Com as duas chaves (1) fechadas e as chaves (2) abertas, os capacitores se carregam como mostra o circuito a seguir (fig 1).

    

     2. Fechando-se as duas chaves (2) e abrindo-se as três chaves (1), os capacitores mantêm a sua carga elétrica o novo circuito está mostrado na figura a seguir:

    

    

     Os dois capacitores idênticos, de capacitância C, em série, têm uma capacitância equivalente igual a C/2. Sendo Q a carga de cada um deles, a associação tem uma carga total igual a Q. Portanto:

     Q = [ C/ 2] . U

     Do gráfico dado tiramos:

     U = 24V

     Q = [( 1,0 . 10−3 . 24)/ 2]

    

Q = 1,2 . 10−2C

     Observações:

  1. O enunciado mencionou três capacitores, quando na realidade são apenas dois.
  2. Devemos entender também que a contagem de tempo tem sua origem (t = 0) a partir do instante em que as chaves (2) foram fechadas e (1) abertas.

     Resposta: C

1.19  Questão 19 - Indução Magnética

     Uma bobina metálica circular de raio r, com N espiras e resistência elétrica R, é atravessada por um campo de indução magnética de intensidade B. Se o raio da bobina é aumentado de uma fração ∆r << r, num intervalo de tempo ∆t, e desconsiderando as perdas, a máxima corrente induzida será de

     Resolução

    

     A corrente elétrica irá surgir nessa bobina devido à variação da área ∆A que é atravessada pelo campo magnético, assim:

     1. Cálculo da variação da área ∆A.

     ∆A = π(r + ∆r)2 − πr2

     ∆A = π(r2 + 2 r ∆r + ∆r2) − πr2

     ∆A = πr2 + π2 r ∆r + π∆r2 − πr2

     ∆A = π2 r ∆r, pois ∆r2 pode ser desprezado.

     2. Na situação de máxima variação de fluxo (∆Φ), temos:

     ∆Φ = N B ∆A

     ∆Φ = N B (π2 r ∆r)

     3. O módulo da força eletromotriz induzida E será dado por:

     |E| = [( |∆Φ|)/( ∆t)]

     E = [( 2 πN B r ∆r)/( ∆t)]

     4. Finalmente:

     i = [ E/ R]

    

i = [( 2 πN B r ∆r)/( R ∆t)]

     Resposta: A

1.20  Questão 20 - Efeito Doppler Relativístico

     Enquanto em repouso relativo a uma estrela, um astronauta vê a luz dela como predominantemente vermelha, de comprimento de onda próximo a 600nm. Acelerando sua nave na direção da estrela, a luz será vista como predominantemente violeta, de comprimento de onda próximo a 400nm, ocasião em que a razão da velocidade da nave em relação à da luz será de

     Resolução

     Para o Efeito Doppler relativístico, temos:

     [ 1/( λobs)] = √{[( 1 ± β)/( 1 ± β)]} . [ 1/( λfonte)]

na qual β é a razão entre o módulo da velocidade do observador (V) e o módulo de velocidade da luz (c):

     β = [ V/ c]

     No caso em que obsevador e fonte se aproximam, temos:

    

     [ 1/( λobs)] = √{[( 1 + β)/( 1 − β)]} . [ 1/( λfonte)]

     [ 1/ 400] = √{[( 1 + β)/( 1 − β)]} . [ 1/ 600]

     [ 3/ 2] = √{[( 1 + β)/( 1 − β)]}

     [ 9/ 4] = [( 1 + β)/( 1 − β)]

     9 − 9 β = 4 + 4 β

     13 β = 5

    

β = [ 5/ 13]

     Resposta: E

     As questões dissertativas, numeradas de 21 a 30, devem ser desenvolvidas, justificadas e respondidas no caderno de soluções

1.21  Questão 21 - Circuitos Elétricos

     No circuito abaixo os medidores de corrente e tensão elétrica são reais, ou seja, possuem resistência interna. Sabendo-se que o voltímetro acusa 3,0 V e o amperímetro, 0,8 A, calcule o valor da resistência interna do voltímetro.

    

     Resolução

     Esquematizando o circuito, temos:

    

     Do enunciado: UAB = 3,0V

     Assim: U2 = R2 i2

     3,0 = 10 i2

     i2 = 0,3 A

     Mas i2 = i3, pois R2 e R3 têm valores iguais.

     A intensidade total da corrente elétrica (i) pode ser determinada por:

     i = i2 + i3 + iv

     0,8 = 0,3 + 0,3 + iv ⇒ iv = 0,2A

     Portanto:

     Uv = Rv . iv

     3,0 = Rv . 0,2

    

Rv = 15 Ω

     Resposta: 15 Ω

1.22  Questão 22 - Mecânica, Cinemática

     No tráfego, um veículo deve se manter a uma distância segura do que vai logo à frente. Há países que adotam a "regra dos três segundos", vale dizer: ao observar que o veículo da frente passa por uma dada referência ao lado da pista, que se encontra a uma distância d, o motorista deverá passar por essa mesma referência somente após pelo menos três segundos, mantida constante sua velocidade v0. Nessas condições,

     1. supondo que o veículo da frente pare instantaneamente, estando o de trás a uma distância ainda segura de acordo com a "regra dos três segundos", calcule o tempo T da frenagem deste para que ele possa percorrer essa distância d, mantida constante a aceleração.

     2. para situações com diferentes valores da velocidade inicial v0, esboce um gráfico do módulo da aceleração do veículo de trás em função dessa velocidade, com o veículo parando completamente no intervalo de tempo T determinado no item anterior.

     3. considerando que a aceleração a depende principalmente do coeficiente de atrito μ entre os pneus e o asfalto, explique como utilizar o gráfico para obter o valor máximo da velocidade vM para o qual a "regra dos três segundos" permanece válida. Sendo μ = 0,6 obtenha este valor.

     Resolução

     A distância d deve ser percorrida com velocidade de módulo V0 em 3s. Portanto:

     d = 3 V0 (SI)

     1) Usando a equação da velocidade escalar média:

     [( ∆s)/( ∆t)] = [( V0 + Vf)/ 2]

     [ d/ T] = [( V0 + 0)/ 2] ⇒ [( 3 V0)/ T] = [( V0)/ 2]

     2) V = V0 + γt

     0 = V0 − a . 6,0 ⇒ a = [( V0)/ 6,0] (SI)

    

     3) PFD: Fat = m a

     μm g = m a

    

⇒ a = μg = 6,0 m/s2

     Sendo a = [( V0)/ 6,0] vem:

     6,0 = [( VM)/ 6,0]

     ⇒ VM = 36,0 m/s

     Respostas:

     1) T = 6,0s

     2) a = [( V0)/ 6,0] (SI)

     3) VM = 36,0m/s

1.23  Questão 23 - Mecânica, Hidrodinâmica

     Um cilindro vertical de seção reta de área A1, fechado, contendo gás e água é posto sobre um carrinho que pode se movimentar horizontalmente sem atrito. A uma profundidade h do cilindro, há um pequeno orifício de área A2 por onde escoa a água. Num certo instante a pressão do gás é p, a massa da água, Ma e a massa restante do sistema, M. Determine a aceleração do carrinho nesse instante mencionado em função dos parâmetros dados. Justifique as aproximações eventualmente realizadas.

     Resolução

    

     1) Aplicando-se a Equação de Bernoulli entre A e B, vem:

     PA + [( μVa2)/ 2] + μg h = patm + [( μV2)/ 2]

     Nota: admitimos que o orifício será feito próximo ao fundo do recipiente e vamos considerar Va ≅ 0.

     p + μg h = patm + [( μV2)/ 2]

    

V2 = [( 2 (p − patm))/( μ)] + 2 g h (1)

     2) Teorema do impulso:

     I = F ∆t = (∆m) V

     ∆m = μA2 ∆x

     [( ∆m)/( ∆t)] = μA2 [( ∆x)/( ∆t)], em que [( ∆x)/( ∆t)] = V

     [( ∆m)/( ∆t)] = μA2 V

     Da qual:

     F = [( ∆m)/( ∆t)] . V = μA2 V . V

    

F = μA2 V2

     3) 2a Lei de Newton:

     F = (Ma + M) a

     μA2 V2 = (Ma + M) a

     a = ( [( μA2)/( Ma + M)] ) V2 (2)

     De (1) em (2), vem:

     a = ( [( μA2)/( Ma + M)] ) [ [( 2 (p − patm))/( μ)] + 2 g h ]

     a = [( μA2)/( Ma + M)] [( 2 (p − patm + μg h))/( μ)]

     Resposta:

    

a = [( 2 A2 (p − patm + μg h))/( Ma + M)]

     Observação:

     O enunciado não citou a pressão atmosférica (patm), a densidade da água (μ) e o módulo g da aceleração da gravidade. Aproximações feitas:

     1) O nível da água mantém-se horizontal.

     2) O orifício próximo ao fundo do recipiente.

     3) Velocidade nula na superfície da água.

1.24  Questão 24 - Ondulatória, Acústica

     Um dado instrumento, emitindo um único som de frequência f0, é solto no instante t = 0 de uma altura h em relação ao chão onde você, imóvel, mede a frequência f que a cada instante chega aos seus ouvidos. O gráfico resultante de [ 1/ f] ×t mostra uma reta de coeficiente angular −3,00 ×10−5. Desprezando a resistência do ar, determine o valor da frequência f0.

     Resolução

     Gráfico qualitativo do fenômeno

    

     Equação do Efeito Doppler sonoro:

     [ f/( Vsom + Vobservador)] = [( f0)/( Vsom − Vfonte)]

     [ f/( 340 + 0)] = [( f0)/( 340 − 10 t)]

     f = [( 340 f0)/( 340 − 10 t)] ⇒ [ 1/ f] = [ 1/( f0)] − [ t/( 34 f0)]

     Equação da reta: y = b + ax

     O coeficiente angular (a) da reta (tanα no gráfico da figura) corresponde a:

     a = [ 1/( 34 f0)] ⇒ f0 = [ 1/ 34 a]

     Sendo a = 3,00 . 10−5 (unidades SI), vem:

     f0 = [ 1/( 34 . 3,00 . 10−5)] (Hz)

     Da qual:

    

f0 ≅ 980,4 Hz

     Resposta: Aproximadamente 980,4 Hz

1.25  Questão 25 - Mecânica, Atrito

     Dois garotos com patins de rodinhas idênticos encontram-se numa superfície horizontal com atrito e, graças a uma interação, conseguem obter a razão entre seus respectivos pesos valendo-se apenas de uma fita métrica. Como é resolvida essa questão e quais os conceitos físicos envolvidos?

     Resolução

     Representação do contexto proposto:

    

     (I) Teorema da energia cinética:

     τ = [( m V2)/ 2] − [( m V02)/ 2] ⇒ − Fat d = − [( m V02)/ 2]

     μC m g d = [( m V02)/ 2]

    

⇒ V0 = √{2 μC g d} (1)

     (II) Conservação da quantidade de movimento no ato do mútuo empurrão:

     Qf = Qi ⇒QA +QB = 0 ⇒QA = −QB

     Em módulo: QA = QB ⇒ mA VA = mB VB

     mA g VA = mB g VB ⇒ PA VA = PB VB

    

[( PA)/( PB)] = [( VB)/( VA)] (2)

     (III) Substituindo-se (1) em (2):

     [( PA)/( PB)] = [( √{2 μC g dB})/( √{2 μC g dA})]

     Da qual:

     [( PA)/( PB)] = √{[( dB)/( dA)]}

     Utilizando-se a fita métrica, medem-se as distâncias percorridas pelos garotos até sua imobilização e, por meio da expressão acima, determina-se a relação entre seus pesos.

     Resposta: Foram utilizados o teorema da energia cinética (ou princípio de conservação da energia mecânica) e o princípio de conservação da quantidade de movimento.

1.26  Questão 26 - Termologia, Conversão de Energia

     Considere uma garrafa térmica fechada contendo uma certa quantidade de água inicialmente a 20oC. Elevando-se a garrafa a uma certa altura e baixando-a em seguida, suponha que toda a água sofra uma queda livre de 42 cm em seu interior. Este processo se repete 100 vezes por minuto. Supondo que toda a energia cinética se transforme em calor a cada movimento, determine o tempo necessário para ferver toda a água.

     Resolução

    

     n Epot = Q

n m g h = m c ∆θ

     n = [( c ∆θ)/ g h]

     n = [( 4200 . (100 − 20))/ 10 . 0,42]

     n = [ 4200 . 80/ 4,2]

    

n = 80000 quedas

     Para calcular o tempo ∆t para a fervura da água, vem:

     100 quedas --- 1,0 minuto

     80000 quedas --− ∆t

     100 ∆t = 800

    

∆t = 800 minutos

     Resposta: 800 minutos

1.27  Questão 27 - Resistência Elétrica Variável

     Considere superpostas três barras idênticas de grafite com resistividade ρ = 1,0 ×10−4 Ωm, 15 cm de comprimento e seção quadrada com 2,0 em de lado. Inicialmente as três barras têm as suas extremidades em contato com a chapa ligada ao contato A. Em seguida, a barra do meio desliza sem atrito com velocidade constante v = 1,0 cm/s, movimentando igualmente o contato B, conforme a figura. Obtenha a expressão da resistência R medida entre A e B como função do tempo e esboce o seu gráfico.

    

     Resolução

     Seja A a área da seção transversal:

     A = (2,0 cm)2 = (2,0 . 10−2m)2 = 4,0 . 10−4 m2

     O comprimento L da barra é:

     L = 15 cm = 15 . 10−2 m

     Então a resistência R de cada barra é dada pela 2a Lei de Ohm:

     R = ρ[ L/ A]

     R = 1,0 . 10−4 [( 15 . 10−2)/( 4,0 . 10−4)] (unidades SI)

    

R = 3,75 . 10−2

     Para t = 0, as três barras superpostas são equivalentes a três resistores em paralelo (fig. 1)

    

     R0 = [ R/ 3]

     R0 = [( 3,75 . 10−2)/ 3] Ω

    

R0 = 1,25 . 10−2

     A barra do meio desliza com velocidade constante V = 1,0 cm/s e percorre os 15 cm de comprimento num intervalo de tempo de 15 s.

     Assim, para t = 15 s teremos a situação da figura (2):

    

     Essa situação é equivalente a:

    

     Rf = [ R/ 2] + R = [ 3R/ 2]

     Rf = [( 3 ×3,75 . 10−2)/ 2] (Ω)

     ⇒ Rf ≅ 5,62 . 10−2

     Como a barra do meio foi deslizada com velocidade escalar constante, podemos concluir que a variação da resistência equivalente obedece a uma função de 1o grau em t. Assim, temos o gráfico da figura 4.

    

     Do gráfico, obtemos o coeficiente angular da reta:

     m = tanα = [( (5,62 − 1,25) . 10−2 Ω)/ 15s]

m ≅ 0,29 . 10−2 Ω/s

     A equação dessa reta é a função procurada:

     R = R0 + m . t

     R = 1,25 . 10−2 + 0,29 . 10−2 t (unidades SI)

1.28  Questão 28 - Mecânica, Choques Elásticos e Eletrostática

     Na ausência da gravidade e no vácuo, encontram-se três esferas condutoras alinhadas, A, B e C, de mesmo raio e de massas respectivamente iguais a m, m e 2m. Inicialmente B e C encontram-se descarregadas e em repouso, e a esfera A, com carga elétrica Q, é lançada contra a intermediária B com uma certa velocidade v. Supondo que todos movimentos ocorram ao longo de uma mesma reta, que as massas sejam grandes o suficiente para se desprezar as forças coulombianas e ainda que todas as colisões sejam elásticas, determine a carga elétrica de cada esfera após todas as colisões possíveis.

     Resolução

     Colisão entre A e B: há troca de velocidade (colisão frontal e perfeitamente elástica entre corpos de mesma massa).

    

     A carga Q de A se divide em Q/2 para A e Q/2 para B (eletrização por contato entre esferas iguais).

     Colisão entre B e C: A carga Q/2 de B se divide em Q/4 para B e Q/4 para C. Cálculo das velocidades de B e C após a colisão:

    

     e = [( vel. rel. depois)/( vel. rel. antes)]

     1 = [( v2 − v1)/ v] ⇒ v2 − v1 = v (1)

     Qantes = Qdepois

mv = mv1 + 2mv2

     v = v1 + 2 v2 (2)

     De (1) e (2):

     v2 = [ 2v/ 3]

     v1 = − [ v/ 3]

     A esfera B volta após o choque com a esfera C e colide novamente com A. Entre A e B, ocorre eletrização por contato e suas cargas elétricas passam a ser:

     [( [ Q/ 2] + [ Q/ 4])/ 2] = [ 3Q/ 8]

     Assim, A e B ficam com cargas iguais a [ 3Q/ 8] e C fica com carga [ Q/ 4].

     Pelo princípio de conservação das cargas elétricas, temos:

     [ 3Q/ 8] + [ 3Q/ 8] + [ Q/ 4] = Q

     Resposta:

     a) [ 3Q/ 8]

     b) [ 3Q/ 8]

     c) [ Q/ 4]

1.29  Questão 29 - Mecânica, Movimento Harmônico Simples

     Um sistema mecânico é formado por duas partículas de massas m conectadas por uma mola, de constante elástica k e comprimento natural 2 l0, e duas barras formando um ângulo fixo de 2 α, conforme a figura. As partículas podem se mover em movimento oscilatório, sem atrito, ao longo das barras, com a mola subindo e descendo sempre na horizontal. Determine a frequência angular da oscilação e a variação ∆l = l0l1, em que l1 é o comprimento da mola em sua posição de equilíbrio.

    

     Resolução

     Cada partícula realiza um MHS na direção da barra.

    

     Aplicando o princípio fundamental da Dinâmica, temos:

     Fel . sen α+ m g cosα = m . a

     k . ∆l. sen α+ m g cosα = m a (1)

     De acordo com a fig. 2, vem:

    

sen α = [( ∆l)/ 2x] ⇒ ∆l = 2x sen α

     Logo: k . 2x sen α. sen α+ m g cosα = ma

     a = [( 2k . sen2 α)/ m] . x + g cosα

     a = A . x + B,

com A e B constantes, e A = ω2.

     A equação a = Ax + B é característica do MHS.

     Portanto:

     ω2 = [( 2 k sen2 α)/ m]

     ω = √{[( 2 k sen2 α)/ m]}

     ω = sen α√{[ 2 k/ m]}

     Na posição de equilíbrio, fazendo a = 0 na equação (1), vem:

     k . ∆lsen α = − m g cosα

    

|∆l| = [( m g cotα)/ k]

     Respostas:

     ω = sen α. √{[ 2k/ m]}

     |∆l| = [( m g cotα)/ k]

     Obs.: Consideramos o comprimento natural da mola igual a l0.

1.30  Questão 30 - Teoria dos Circuitos

     No circuito da figura o capacitor encontra-se descarregado com a chave A aberta que, a seguir, é fechada no instante t1, sendo que o capacitor estará totalmente carregado no instante t2. Desprezando a resistência da bateria V, determine a corrente no circuito nos instantes t1 e t2.

    

     Resolução

     No instante t1, quando a chave é fechada, o capacitor entra em processo de carga. Nesse instante, atuará como um curto-circuito para o resistor que está associado em paralelo com ele, assim:

     Instante t1:

     i1 = [ V/ R]

     No instante t2, com o capacitor plenamente carregado, ele atua como circuito aberto, ou seja, não é percorrido por corrente elétrica, assim:

     Instante t2:

     i2 = [ V/( Req)]

     i2 = [ V/ 2R]

     Respostas: i1 = [ V/ R]; i2 = [ V/ 2 R]